Preparation

¡Supera tus tareas y exámenes ahora con Quizwiz!

A chin fissure is a dominant trait. If a father who is homozygous-dominant for this trait and a mother who is homozygous-recessive for this trait mate, what are the chances that their first child will have a chin fissure? 25% 75% 0% 50% 100%

100% Because the father is homozygous-dominant, it can be inferred that he possesses a dominant gene pair for the chin fissure trait (FF). On the other hand, the mother is homozygous-recessive for this trait; therefore, phenotypically she would have a "normal chin" because she has an identical gene pair that does not code for a chin fissure (ff). Each child would receive an allele from each parent, but the pair of genes would not be identical (this is termed heterozygous (Ff)). Because the children would inherit a dominant form of the allele from their father, they would all have the appearance of a chin fissure.

You are fitting your patient's right eye with a toric soft contact lens. The patient's manifest refraction is -2.00 -1.50 x 095. You apply a -1.75 -1.25 x 085 diagnostic toric soft contact lens. It fits well, but the prism base down marking consistently locates halfway between the 6 o'clock and 7 o'clock positions. What axis should you order? 100 degrees 110 degrees 080 degrees 070 degrees 095 degrees

110 degrees Applying LARS (Left Add, Right Subtract) to compensate for lens rotation, because the lens is rotated to the Left, you would Add the amount of the left rotation to the manifest refraction axis. Every hour on the clock dial would translate to 30 degrees rotation. In the above example, the lens is rotated to the doctor's left by 15 degrees (between the 6 and 7 o'clock hours). Add the amount of rotation (15 degrees) to the cylinder axis of the MANIFEST refraction (095 degrees). This results in a cylinder axis of 110 degrees to order.

A person can just barely detect the difference between two weights; one weighs 12 pounds and the other weighs 10 pounds. What is this person's just noticeable difference for a weight of 70 pounds? 2 pounds 14 pounds 8 pounds 56 pounds 20 pounds

14 pounds Weber's Law deals with the just noticeable difference (JND) and can be expressed mathematically as: K= delta I/I, where K= Weber's constant, delta I= the difference threshold, and I= the original stimulus intensity (weight etc.) For the above example, we must first solve for Weber's constant: 12-10/10= 0.2. Using this Weber's constant we can then solve for the just noticeable difference, or the increment threshold for 70 pounds. X-70/70= 0.2, X= 14 pounds. Therefore, the above person will just be able to discern the difference between a 70-pound weight and an 84-pound weight.

What is the equivalent of a reduced Snellen (RS) 20/50 optotype in metric (M) notation (assuming a working distance of 40 cm)? 2M 0.67M 0.5M 1M

1M To convert from reduced Snellen (RS) to metric (M) notation, one must divide the denominator by 50. In the above example 50/50 = 1M. To convert from Reduced Snellen to Printer's point, divide the denominator by 6. To convert from Printer's point to metric, divide by 8. To convert from metric notation to reduced Snellen, multiply by 50; this will give you the denominator of the reduced Snellen equivalent. A good rule of thumb is 1M = RS 20/50 = 8 point. attached is the conversion triangle:

A patient who has a high spatial frequency cut-off of 40 cycles per degree will have what predicted Snellen visual acuity? 20/40 20/20 20/30 20/15

20/15 In order to convert from cycles per degree to Snellen acuity, simply divide 600 by the cycles per degree; this will solve for the denominator of the Snellen acuity. For the above example 600/40 = 15. Therefore, the predicted Snellen acuity would be 20/15.

What is the power (in diopters) of a concave mirror located in air with a radius of curvature of 20 cm? +5.00 D -10.00 D +10.00 D -5.00 D

+10.00 D A concave mirror converges light and therefore acts like a convex lens, hence concave mirrors have positive dioptric values, whereas convex mirrors diverge light and possess negative dioptric powers. The equation used to determine the power of a mirror is P= -2n/r where P= the power of the mirror in diopters, n= the index of refraction of the surrounding medium, and r= the radius of curvature of the mirror in meters. Unless otherwise stated, always assume that the index of refraction of air is 1.0. Input the values from the above question: P= -2(1)/-0.2= +10.00 D.

A patient is using a +16D stand magnifier with a +2.00 add bifocal. If the distance separating the two lenses is 25 cm, what is the equivalent power of this combination? +26D +22D +10D +18D

+10D De= D1+D2 -tD1D2 De= equivalent power; D1= power of magnifier; D2= power of add; t= separation of lenses (in meters) De = (16+2) - 0.25(16)(2) De= 18-8 = 10D

A convex mirror in water (n= 1.33) has a radius of curvature of 12 cm. What is the dioptric power of the mirror? -11.08 D -22.17 D +8.33 D +22.17 D

-22.17 D A concave mirror converges light and therefore acts like a convex lens, hence concave mirrors have positive dioptric values. Conversely, convex mirrors diverge light and possess negative dioptric powers. The equation used to determine the power of a mirror is P= -2n/r, where P= the power of the mirror in diopters, n= the index of refraction of the surrounding medium, and r= the radius of curvature of the mirror in meters. P= -2(1.33)/0.12 = -22.17 D. Remember, a convex mirror will have a positive radius of curvature and a concave mirror will have a negative radius of curvature.

What is the front surface power of a lens in air that has a refractive index of 1.50 and radius of curvature of 50 cm? 3.00 D 2.00 D 1.00 D 1.50 D

1.00 D To solve this problem, input the values into the equation for a single surface power, F = n'-n/r F= the power of the lens n'= the index of the medium that light is entering (the lens) n= the index of the medium in which light is exiting (medium surrounding the lens; in this case, air) r= the radius of curvature (in meters) of the lens. Solve for F = 1.50-1.0/0.5 = 1.00 D.

A patient returns to your office reporting that her eyes feel strange when she is studying. You notice that her eyes are positioned 6 mm below the optical centers of her new glasses when she reads. The prescription in her right eye is -4.00 DS and -7.00 DS in her left eye. How much vertical prism is induced when this patient reads at near? 6.6 prism diopters base down 4.2 prism diopters base down 1.8 prism diopters base down 2.4 prism diopters base down

1.8 prism diopters base down Use the Prentice rule to solve this problem: prism diopters (pd) = dxF, where d= the distance from the optical center (in centimeters) and F= the power of the lens (in diopters). In this instance, the patient is looking through base down prism in both eyes, which will cancel some of the prismatic effect because the bases are aligned. Solving for the amount of prism induced in the right eye: pd= 0.6(-4.00) = 2.4 base down prism. Also, solve for the left eye: pd= 0.6(-7.00) = 4.2 base down prism. Now, subtract the two values to determine the total prismatic effect experienced by the patient: 4.2-2.4= 1.8 base down prism. Alternatively, you can omit one of the steps by initially determining the total power difference in the vertical meridian between the two lenses, which is 3.00 D (7-4= 3). Then you can multiply this power difference by the distance between the patient's line of sight and the optical center, which is 6 mm in this question. Pd= 0.6(3) = 1.8 prism diopters base down over the left eye. Generally, vertical imbalances of smaller magnitudes do not pose too much of a problem for single vision lenses, as the patient can tilt her head to re-align the optical centers with her line of sight, thus eliminating any possible diplopia.

When performing a unilateral cover test on your patient, you note the following: upon covering the right eye, the left eye moves in 1/10 times. Upon covering the left eye, the right eye moves in 4/10 times. The alternating cover test measures 25 prism diopters base-in. What is your diagnosis based on these findings? 25 prism diopter intermittent alternating exotropia; right eye preferred 25 prism diopter intermittent alternating esotropia; left eye preferred 25 prism diopter intermittent alternating esotropia; right eye preferred 25 prism diopter intermittent alternating exotropia; left eye preferred

25 prism diopter intermittent alternating exotropia; left eye preferred The unilateral cover test will tell you the eye (or eyes) affected, the direction, and the frequency of the ocular deviation. In this case, the eyes lose fixation 5/10 times, which shows that the frequency is intermittent (it would be constant if at least 1 eye moved 10/10 times). Since each eye moves at least once when the other is covered, the deviation is considered to be alternating. The uncovered eye is noted to move "in" on unilateral cover test, meaning that the deviation is an exotropia (if the eye moves "out" it is an esotropia). Since the right eye loses fixation more than the left eye (4/10 vs. 1/10), the left eye is considered to be the preferred eye. Also, the alternating cover test will tell you the full amount of the deviation, which is 25 prism diopters in this case.

A 6-foot tall man wishes to purchase a plane mirror in which he can visualize his whole length at the same time. How tall must the mirror measure in order for the above to occur? 5.2 feet tall 3 feet tall 6 feet tall 2.3 feet tall 4.5 feet tall

3 feet tall In order for a person to see his entire reflection, a plane mirror must be half as tall as the person. This holds true regardless of the position of the person. For the above example, 6/2= 3 feet.

A patient brings in an old pair of glasses to your office and asks how much prism is in the lenses. With the lensometer, you measure 7 prism diopters base up and 4 prism diopters base out in the right eye, and 4 prism diopters base up and 5 prism diopters base out in the left eye. What is the total amount of prism in the glasses? 11 prism diopters base up and 1 prism diopter base out 11 prism diopters base up and 9 prism diopters base out 3 prism diopters base up in the right eye and 9 prism diopters base out 3 prism diopters base up in the right eye and 1 prism diopter base out

3 prism diopters base up in the right eye and 9 prism diopters base out For vertical prism, if the bases are oriented in the same direction, they will cancel each other out. When the bases are oriented in opposite directions in the vertical meridian (i.e., base up and base down), the powers will add together. The opposite holds true for prisms with their bases oriented horizontally. If the prism bases are both base out or both base in, the powers are additive, while if they are opposite (that is, one is base in and one is base out), the powers will cancel.

What is the NET amount of moles of ATP that is produced by the electron transport chain (not including glycolysis)? 30 moles of ATP 2 moles of ATP 34 moles of ATP 38 moles of ATP 6 moles of ATP

34 moles of ATP The electron transport chain yields a total of 34 moles of ATP. Glycolysis produces a total of 2 moles of ATP. The overall net of cellular respiration is 36 moles of ATP.

What separation distance will make the combination of a +3.00 D thin lens and a +10.00 D thin lens afocal? 1.7 cm 2.3 cm 23 cm 17 cm 43 cm 0.43 cm

43 cm For this question, the equation for equivalent power of a thick lens system should be used; solving for thickness (t). De = D1 + D2 - (t/n) x D1D2 De = equivalent power, D1 = front surface power, D2 = back surface power t = thickness of lens system, n = index between the 2 surfaces An afocal system has its focal points (F and F') located at infinity. Therefore, an incident parallel pencil of light rays will emerge into image space as a parallel pencil as well. Another way to characterize an afocal system is that the equivalent power (De) is 0. In the above question, De = 0, D1 = +3.00, D2 = +10.00, n = 1 0 = 3 + 10 - ((t/1) x (3) x (10)) 0 = 13 - (t x 3 x 10) 0 = 13 - 30t 30t = 13 t = 0.43 m (or 43 cm) If the two lenses are separated by 43 cm, the lens system can be considered afocal. This type of combination of two plus lenses is also an example of a simple astronomical (Keplerian) telescope. Keep in mind that the image in this type of optical system is inverted.

What is the total cumulative dosage of chloroquine that is MOST commonly associated with retinal toxicity? 1,000 grams 1500 grams 100 grams 4,000 grams 460 grams

460 grams The amount of chloroquine that is usually required to produce retinal toxicity is a total cumulative dosage of 460 grams. Symptoms of chloroquine toxicity typically involve decreased vision, abnormal color vision, and difficulty dark-adapting. Retinal evaluation commonly reveals a bull's-eye appearance of the macula in which there is a ring of depigmentation surrounded by a ring of increased pigmentation. Foveal reflex is also diminished in patients presenting with this condition. It is also possible to develop retinal toxicity as a result of hydroxychloroquine use, but this is much less common than with chloroquine. Typically, a cumulative dose of 1,000 grams is required to produce clinical signs and symptoms.

A Galilean telescope has an ocular lens with a power of -32.00 D and an objective lens with a power of +8.00 D. What is the magnification provided by the telescope? 0.25x 256x 4x 8x

4x To calculate the magnification (M) of a telescope, divide the power of the ocular lens (Doc) by the power of the objective lens (Dobj): M= -Doc/Dobj. In the example above, M= -(-32 D)/8 D= 4x. The magnification of a Galilean telescope is positive due to the fact that its ocular has a minus powered lens. The magnification of an astronomical telescope is negative and therefore its image will be upside down.

What wavelength of light is most readily absorbed by the photopigment rhodopsin? 555 nm 480 nm 507 nm 460 nm

507 nm The absorption spectrum curve for rhodopsin shows peak absorption for wavelengths that are roughly 507 nm. This is determined by taking a sample of rhodopsin and projecting a fixed quantity of monochromatic light onto it and measuring the amount of light that is transmitted. Light that is not transmitted is absorbed; therefore the absorption curve and the transmitted curve are reciprocals of each other. The procedure is repeated with many different wavelengths. The wavelength that results in the greatest amount of absorption (or the least amount of transmitted light) is obviously the one that has the highest probability of absorption by rhodopsin. (555 nm is the peak sensitivity for photopic conditions.)

A 2.5x Galilean telescope has a -25D ocular lens. When focused for infinity, what is the length of the telescope? 4 cm 6 cm 10 cm 5 cm 14 cm

6 cm First, use the formula below to solve for the power of the objective lens: M= -Doc/Dobj Doc= power ocular; Dobj= power objective; t= separation of lenses 2.5= - (-)25/Dobj Dobj= 10D Then, use the equation below to calculate the separation of the lenses: t= f'obj + f'oc f'obj= 1/10D = 0.10 m f'oc = 1/-25D = 0.04 m t= 0.10 + -0.04 = 0.06 m or 6 cm 10 cm - incorrect- would come up with this answer if only took in account the focal length of the objective lens 4 cm - incorrect- would come up with this answer if only took in account the focal length of the ocular lens

Tear volume in a normal, healthy, young adult measures approximately between which of the following values? 17.0 - 20.0 microliters 2.0 - 5.0 microliters 9.0 - 12.0 microliters 6.0 - 8.0 microliters 13.0 - 16.0 microliters

6.0 - 8.0 microliters Tear volume has been measured by several methods to be approximately 6-8 microliters in normal individuals, with lesser values occurring in conditions of aqueous tear deficiency. This has implications for drug delivery, since the normal ophthalmic drop volume varies between 25 and 50 microliters, effectively overwhelming the native tear film upon instillation.

While performing the astigmatic clock dial, your patient reports that the clearest/blackest line is the 2-8 line, while the 5-11 line is the least clear. What would be the corresponding axis of astigmatism? 180 degrees 30 degrees 60 degrees 150 degrees

60 degrees In order to determine the corresponding axis of astigmatism utilizing the clock dial, one must multiply the smallest number of the clearest clock position by 30 degrees. In our case 2 x 30= 60 degrees. In general the line perpendicular to the clearest line is generally the least clear, as this corresponds to the second principal meridian of the eye. Remember the principal meridians of the eye are 90 degrees apart.

What is the minimum thickness necessary for an antireflective coating (n= 1.9) to be useful against incident light of 530 nm wavelength? 278.9 nm 69.7 nm 139.5 nm 58.3 nm 132.5 nm

69.7 nm The equation for finding the minimum antireflective coating thickness is: thickness = wavelength/(4 x index of coating) thickness = 530 / (4 x 1.9) thickness = 530 / 7.6 thickness = 69.73 nm

Which one of the following bitoric GP contact lenses would NOT induce cylinder if rotated to a misaligned position on the eye? 7.54 mm / +1.50 D --------------------- 7.99 mm / +2.75 D 7.63 mm / -1.50 D --------------------- 8.11 mm / +1.12 D All of the options listed would induce cylinder if rotated off axis 7.46 mm / -4.25 D --------------------- 8.13 mm / -1.75 D 7.58 mm / -5.37 D --------------------- 8.18 mm / -0.50 D

7.63 mm / -1.50 D --------------------- 8.11 mm / +1.12 D Cylinder power effect (CPE) bitoric and base curve toric (with a spherical front-surface) gas permeable (GP) lenses will induce unwanted cylinder if the lens rotates off axis. The resulting cylinder is due to cross-cylinder effects. However, a spherical power effect (SPE) bitoric will not induce unwanted cylinder regardless of lens rotation. To determine whether a GP lens is a spherical power effect (SPE) or cylinder power effect (CPE) bitoric, measure the two base curves using a radiuscope and the two raw contact lens powers using a lensometer. If the difference between the two base curve meridians (in diopters) is the same as the difference between the two raw powers, the lens is an SPE bitoric. This is the case for only one of the above answers. Converting mm of base curve radius to diopters results in 7.63 mm = 44.25 D and 8.11 mm = 41.62; a difference of 2.62 D. The difference between the two raw powers of +1.12 D and -1.50 D is also 2.62 D. Therefore, this lens is a spherical power effect (SPE) bitoric GP contact lens.

Oral acyclovir is most effective for patients presenting with ocular findings associated with herpes zoster if administered within which of the following periods following the onset of the disease? 4-5 days 7-10 days 24 hours 10-12 hours 72 hours

72 hours Oral acyclovir is the mainstay of therapy for patients diagnosed with herpes zoster ophthalmicus. This systemic treatment is maximally beneficial if it is initiated within 72 hours from the onset of the disease (usually the appearance of eyelid lesions). The use of oral acyclovir typically results in quick resolution of skin vesicles, decreases the amount of pain the patient experiences, and reduces the duration of viral shedding and appearance of new lesions. Acyclovir has also been shown to significantly reduce the incidence of ocular findings such as episcleritis, keratitis, and iritis. The recommended dosage is 800mg orally 5 times per day for 7-10 days.

What is the Interval of Sturm for a spherocylindrical lens with a power of +6.00 -2.00 x 090? 25 cm 20 cm 41.7 cm 8.3 cm 16.7 cm

8.3 cm The powers in each meridian of the lens are +6.00 and +4.00. The Interval of Sturm is simply the distance between the focal point of each power. 1/+6.00 = 16.7cm 1/+4.00 = 25.0cm 25cm - 16.7cm = 8.3cm

Which of the following conditions could result in amblyopia due to deprivation? A child born with a large congenital cataract in one eye only A child born with a monocular .5 mm ptosis A three-year-old with a constant right 30 prism diopter esotropia A five-year-old with an uncorrected prescription of OD: +7.00DS and OS: +0.50DS

A child born with a large congenital cataract in one eye only Form deprivation amblyopia results when a clear and focused retinal image is blocked by some type of opacity or obstruction during the critical period. This can occur due to a significant congenital cataract in one eye, a large ptosis that covers most or all of the pupil, or by some other element that occludes the eye. The lack of visual information to the retina causes the other eye (non-occluded eye) to become dominant and have stronger and a greater number of synaptic connections to the brain. Amblyopia causes a disproportionate amount of cortical neurons to respond preferentially to the non-deprived eye. The occlusion must occur during the critical period, and the earlier the occlusion is detected and removed, the better the prognosis. A small ptosis (i.e. 2 mm) would not be expected to cause amblyopia because the pupil would not be occluded. An unequal prescription such as the one in the above question would cause anisometropic amblyopia in which one eye would receive a clear image while the other would receive a blurry image. The brain would favor the clear retinal image, resulting in a strong dominance of cortical neurons for the least ametropic eye. Strabismus results in the perception of two images that are not fusible by the brain, causing diplopia. In order to eliminate double vision, the brain will suppress an eye (usually the deviated eye). This suppression leads to amblyopia.

A person who is missing the photopigment chlorolabe can be classified as which of the following? A protanope A deuteranope A tritanope A protanomalous trichromat A deuteranomalous trichromat

A deuteranope There are several classifications of color-vision defects; hereditary defects are the most common. The two broad categories are dichromacy and anomalous trichromacy. In dichromacy, one of the photopigments is missing; the type of dichromacy is categorized based on which photopigment is lacking. A deuteranope is missing chlorolabe, a tritanope is missing cyanolabe, and a protanope is missing erythrolabe. It is theorized that the missing photopigment is replaced by the photopigments that are present; otherwise, the person would likely suffer a deficit in visual acuity. Anomalous trichromats are in possession of all three photopigments but the absorption spectrum of one of the pigments has been shifted. For a protanomalous trichromat, the spectrum for erythrolabe is shifted towards the shorter wavelengths. A deuteranomalous trichromat displays a shift of the maximum sensitivity of chlorolabe towards the longer wavelengths. Protans and deutans are said to be red-green colorblind, while tritans tend to mix up blues and yellows and are said to possess a blue-yellow defect; this is usually acquired rather than hereditary.

Which of the following patients would be considered legally blind? A patient with Best's disease with best-corrected visual acuities of 10/80 OD and 10/100 OS, with normal visual fields OU A myopic patient with uncorrected visual acuities of 20/400 OD and OS, best-corrected acuities of 20/40 OD and OS, with normal visual fields OU A wet macular degeneration patient with best-corrected acuities of 10/120 OD and 10/200 OS, with normal visual fields OU A patient with a total retinal detachment and no light perception OD and best-corrected visual acuity of 8/40 OS due to wet macular degeneration, with normal diameter visual fields OU A retinitis pigmentosa patient with best-corrected visual acuities of 20/20 OD and OS, with 30 degree in diameter visual fields OU

A wet macular degeneration patient with best-corrected acuities of 10/120 OD and 10/200 OS, with normal visual fields OU "Legal blindness" is a term used by the United States government to determine eligibility for disability benefits, rehabilitation services, schooling, low vision devices and tax exemption programs. There are two parts to the definition of legal blindness: 1. Visual acuity of 20/200 or less in the better-seeing eye with the patient's best correction 2. OR limitation of the field of view such that the widest diameter of the patient's visual field in the better-seeing eye subtends an angle no greater than 20 degrees

A 32-year-old female is seen at your office complaining of a recent onset of blurred vision, only at a distance. A thorough case history reveals that she recently began taking a new medication, which you correctly assume has induced myopia. Which of the following medications is MOST likely to be the culprit? Tylenol® (acetaminophen) Omega III fish oil capsules Accutane® (isotretinoin) Tums® (calcium carbonate)

Accutane® (isotretinoin) Isotretinoin, birth control pills, and diuretics, among many other drugs, can cause myopia in some patients. Myopia most likely results from corneal swelling, which steepens the curvature of the cornea. Drugs that cause swelling of the lens, accommodative spasm, or edema of the ciliary body will also result in myopia. A reduction in the dose of the medication or cessation of the offending drug will usually result in the reversal of nearsightedness. Fish oil, Tylenol®, and Tums® have not been shown to have a correlation with transient myopia development.

Many skin anomalies can mimic malignant lesions. Which of the following skin conditions has the HIGHEST risk of becoming malignant? Papilloma Actinic keratosis Seborrhoeic keratosis Cutaneous horn

Actinic keratosis Actinic keratosis is a precursor to squamous cell carcinoma and appears as scaly, dry skin that does not heal. People with skin that is of lighter pigmentation, along with excessive exposure to ultraviolet light tend to be most at risk for development of this condition. Papillomas may take on various forms and may be viral or non-viral in origin. They can commonly be found on the eyelids or surrounding orbital skin. Viral warts tend to grow at an accelerated rate while non-viral papillomas are usually fairly slow to grow. Papillomas can mimic neoplastic growths, so be sure to rule this out while watching carefully for color change, ulceration, lash loss, bleeding, and vascularization. Cutaneous horns or tags are also benign and are likely a form of papilloma but appear to involve more keratin. Treatment is similar to that of a papilloma. Seborrhoeic keratosis is more commonly seen in middle-aged and elderly persons. This benign, epidermal growth is quite superficial and does not extend into the dermis. It appears like a brown plaque that has been stuck onto someone's skin. The borders are very distinct and there may be some elevation. The lesions may be removed if the patient is concerned about cosmesis.

Which of the following BEST describes the reasoning for the need to taper topical ocular corticosteroids when used for prolonged periods of time? Prevent possible secondary ocular infections Decrease the risk of posterior subcapsular cataract formation Minimize risk of adrenal insufficiency due to decreased production of natural cortisol Minimize the risk of developing steroid-induced elevation of IOP Avoid signs and symptoms of rebound ocular inflammation

Avoid signs and symptoms of rebound ocular inflammation When prescribing a topical ocular corticosteroid, the tapering schedule of the medication is just as important as the initial dose. It is imperative not to prematurely discontinue steroid medications and to closely monitor the patient for signs of rebound inflammation during and after the tapering process. The mechanism for developing rebound inflammation stems from the fact that prolonged use of corticosteroids causes a reduction in the amount of mature circulating leukocytic elements. If the steroid is stopped abruptly, the immature cells can proliferate, producing large quantities of antibodies in response to the residual antigen remaining in the tissue. This physiologic action is what causes rebound inflammation. The exaggerated inflammatory response is more commonly associated with systemic steroid use, but prolonged topical ocular therapy (usually longer than 1 month) can also induce this response. A typical steroid taper should begin once the inflammation is completely controlled. There are many different tapering schedules that may be implemented depending on the patient's condition, but classically, the dose of the steroid should be halved for each given time interval. For example, if you prescribed a steroid q.i.d. x 5 days, you could taper to b.i.d. x 5 days, then q.day x 5 days. The longer the treatment period or more frequent the dosage, the longer the taper will be. With long-term oral corticosteroid use, there is a risk of developing adrenal insufficiency if the steroid is not tapered properly. This is due to the fact that the adrenal glands decrease their normal production of natural cortisol when an individual is taking a dose of steroids. Tapering the steroid is important so that the adrenal glands can return to producing habitual levels of cortisol before complete cessation of the oral steroids.

A chiasmal lesion or mass, such as a pituitary tumor, generally causes what type of visual field defect? Bitemporal hemianopsia Binasal hemianopsia Right homonymous hemianopsia Left homonymous hemianopsia

Bitemporal hemianopsia The center of the optic chiasm contains axons of decussating ganglion cells that originate from the nasal retinas, which process temporal visual field information. The lateral portion of the chiasm is comprised of the axons of the temporal aspect of the retinas, which do not cross over. Lesions most commonly occur in the central portion of the chiasm and not the lateral aspects. Any central chiasmal mass or lesion will cause a bitemporal visual field defect that respects the vertical midline.

Which of the following topical glaucoma medications decreases intraocular pressure by increasing uveoscleral outflow? Timolol Dorzolamide Pilocarpine Brimonidine Brinzolamide

Brimonidine Glaucoma medications lower intraocular pressure by either decreasing aqueous production or by increasing aqueous outflow. There are three classes of drugs for which the mechanism of action is increasing aqueous outflow: cholinergic agonists, prostaglandin analogs, and alpha-2 agonists. Cholinergic agonists (such as pilocarpine) work by increasing trabecular outflow, whereas prostaglandin analogs and alpha-2 agonists work by increasing uveoscleral outflow. The other classes of glaucoma medications, such as beta-blockers and carbonic anhydrase inhibitors, work by decreasing aqueous production. It is important to note that alpha-2 agonists, such as Brimonidine (Alphagan®) and Apraclonidine (Iopidine®), have dual mechanisms of action. This class of medication decreases intraocular pressure by both increasing uveoscleral outflow and decreasing aqueous production.

Which of the following types of congenital cataracts are characteristic of galactosemia? Oil droplet opacities Sunflower cataracts Blue dot (Cerulean) opacities Christmas tree cataracts

Central oil droplet opacities are a type of congenital cataract that is associated with galactosemia, a genetic metabolic disorder that affects the body's ability to metabolize galactose properly. Blue dot (Cerulean) opacities are congenital cataracts and are not usually associated with systemic disease but are thought to be due to autosomal dominant mutations in several genes. Christmas tree cataracts are not considered a congenital type of cataract; they are a rare variant of senile cataracts that have a strong association with myotonic dystrophy. Sunflower cataracts are also not considered congenital cataracts and are due to the abnormal deposition of copper in patients suffering from Wilson's disease.

Which of the following terms describes the phenomenon in which a bacterium directs its movement toward a chemical in its environment? Transposition Chemotaxis Phagocytosis Apoptosis

Chemotaxis Many bacteria possess flagella or thread-like appendages which allow for movement. Certain chemicals attract bacteria (chemoattractants), while others repel them (chemorepellents). Chemotaxis refers to the response of the bacteria to either chemoattractants or chemorepellents. In the absence of either of the aforementioned chemicals, bacteria will move in random patterns. Some bacteria possess genes and proteins which allow for the sensing of concentration gradients in their environment. In the presence of a chemoattractant, bacteria will have longer runs in the appropriate direction.

Which of the following is a precursor to steroid hormones such as testosterone? Sphingolipids Phospholipids Triglycerides Cholesterol

Cholesterol Progesterone, aldosterone, testosterone, estradiol and cortisol are all derived from cholesterol. Cholesterol has a unique configuration comprised of four joined cycloalkane rings. Because these hormones are fat-soluble, they readily pass through cell membranes. They diffuse into the blood and are generally bound to carrier proteins, which transport the hormones to their designated target site where they may further undergo processing or transformation. Sphingolipids are important in cell membranes, especially those located in the central nervous system, such as myelin sheath. Sphingolipids contain sphingosine as a backbone and are then further classified depending on which molecules are attached to that backbone, such as ceremides, gangliosides, sphingomyelin, etc. Phospholipids contain a polar and non-polar end, thus making them amphoteric. This property allows for the formation of bilayers (polar ends aligned together and pointed outwards) resulting in the lipid bilayer commonly seen in cell membranes. Phospholipids are generally comprised of a phosphate group, a choline group (polar), and two fatty acid chains (non-polar) attached to glycerol, which serves as the backbone. Triglycerides are comprised of three fatty acid chains attached to a glycerol backbone. Triglycerides are important in long-term energy storage for use by cells.

What is the name of the surgical procedure in which thermal laser burns are placed in the mid-periphery of the cornea in an attempt to steepen the corneal curvature? Photorefractive keratectomy Radial keratotomy Conductive keratoplasty Laser-assisted in-situ keratomileusis Limbal relaxation incisions

Conductive keratoplasty In cases where the corneal curvature needs be steepened in order to correct for refractive error (hyperopia or presbyopia), conductive keratoplasty (CK) can be a viable surgical option. Although this surgical procedure was used more often in earlier years, it is currently not nearly as widely performed as laser-assisted in-situ keratomileusis (LASIK) and photorefractive keratectomy (PRK). In comparison to CK, LASIK and PRK tend to be safe, have long-standing results, and more predictable outcomes. The CK technique involves using a radiofrequency probe to create burns in either one or two concentric rings in the mid-peripheral region of the cornea. These thermal laser burns cause subsequent stromal shrinkage, which results in an increase in the curvature of the cornea. This change in curvature typically decays over time, but the procedure can be repeated if desired. Radial keratotomy is also an older surgical procedure in which a diamond blade is used to create several radial corneal incisions (the number and depth of the incisions depends on the refractive error) in order to flatten the corneal curvature in patients with myopic refractive errors. Limbal relaxation incisions are similar in that arcuate incisions are made on opposite sides of the corneal periphery in the meridian of the "plus" cylinder axis in order to create flattening of the steep meridian (with some smaller steepening of the flat meridian) in an attempt to reduce the amount of corneal astigmatism. Photorefractive keratotomy (PRK) and laser-assisted in-situ keratomileusis (LASIK) are refractive surgery techniques that use an excimer laser to ablate corneal tissue to a certain depth in either the central cornea (to correct myopia) or peripherally (to correct hyperopia).

Antibiotic resistance that is rapidly spread within a population of bacteria is due to which of the following mechanisms? Budding Binary fission Transformation Conjugation

Conjugation Conjugation occurs between a donor (possesses a conjugative plasmid) and recipient bacteria. The donor bacterium initiates contact with the recipient via a sex pilus, allowing for cell-to-cell contact and transfer of DNA. The plasmids often contain genes that encode for toxin production, virulence factors, and antibiotic resistance. Genetic transformation is achieved by very few strains of bacteria and may only occur during certain phases of growth; therefore, rapid antibiotic resistance is not feasible. Budding and binary fission are means of reproduction but are not directly responsible for antibacterial resistance. Genes must have been transferred that code for resistance prior to budding and binary fission in order for the progeny to contain genes that allow for drug resistance.

When examining a patient, a pinpoint spot on the posterior surface of the crystalline lens known as a Mittendorf's dot is observed on biomicroscopy. What is this a remnant of? Pupillary membrane Primary vitreous Tertiary vitreous Hyaloid artery

Hyaloid artery Mittendorf's dot is a remnant of the hyaloid artery and appears as a small pinpoint-like scar on the posterior surface of the lens. Pupillary membrane remnants would be present as strands in front of the lens.

A 42-year-old patient presents to your office reporting symptoms of watering, mild pain, redness, and swelling in the lower medial canthal region of her right eye. You suspect dacryocystitis as the cause of her symptoms. Which of the following procedures is NOT appropriate when further evaluating this possible diagnosis? Digital palpation of the medial canthal area Exophthalmometry Extraocular muscle motility Gram stain and blood agar cultures of discharge Dilation and irrigation of the lacrimal system

Dilation and irrigation of the lacrimal system The evaluation of a patient with suspected dacryocystitis should involve a detailed case history including a discussion of any previous episodes with similar symptoms, or the presence of any concomitant ear, nose, or throat irritation/infection. External examination of the patient should include the application of gentle pressure to the lacrimal sac region in order to attempt to express any discharge from the punctum; this should be done bilaterally. If any discharge can be recovered, a Gram stain or blood agar culture is helpful in determining the type of bacteria present. In addition to these tests, extraocular motility and evaluation for the presence of proptosis should be completed to rule out orbital cellulitis. In atypical, severe, or non-responding cases, a computed tomography scan (CT) should be considered. It is important to remember that probing, dilation, and/or irrigation of the lacrimal system should not be attempted during an acute infection of the lacrimal gland. This may cause the infection to spread to other areas such as the throat.

Patients with a history of homocystinuria are MOST likely to experience crystalline lens subluxation in which of the following directions? A. Upward and outward B. Downward and inward C. Upward and inward D. Downward and outward

Downward and inward Common ocular sequelae that have been associated with a diagnosis of homocystinuria include ectopia lentis (bilateral crystalline lens subluxation), retinal detachment, and secondary glaucoma. In most cases of ectopia lentis associated with homocystinuria, the lens is more likely to be displaced downward and inward (as compared to upward and outward in Marfan's syndrome). Additionally, in homocystinuria, the lens zonules are markedly abnormal, the lens does not accommodate, and up to 1/3 of the cases of lens subluxation eventually completely dislocate into the vitreous or anterior chamber. Due to the severity of systemic and cardiovascular complications associated with homocystinuria (thrombosis and occlusion), patients presenting with ectopia lentis should be screened for this disease using the sodium nitroprusside test to measure homocysteine in the urine.

Which of the following antibiotics is classified as a macrolide? Tetracycline Tobramycin Amoxicillin Erythromycin

Erythromycin Erythromycin and azithromycin belong to a class of drugs called macrolides. Macrolides are effective antibiotics because they bind to the 50S subunit of bacterial ribosomes, thus interfering with bacterial protein synthesis. Tetracycline and doxycycline are classified as tetracyclines. This class of drugs also interferes with protein synthesis via binding to the 30S ribosomal subunit. Amoxicillin, Augmentin®, cloxacillin, dicloxacillin are common anti-bacterials belonging to a class of drugs termed penicillins. Penicillins disrupt cell wall synthesis, making them valuable and commonly used antibiotics. Tobramycin, gentamicin and neomycin are categorized as aminoglycosides, which serve as antimicrobials via two mechanisms. Aminoglycosides inhibit bacterial protein synthesis and cause the creation of openings in bacterial cell membranes, allowing for increased antibiotic uptake.

A cranial nerve VI palsy will cause what type of ocular deviation? Esodeviation worse with near viewing Esodeviation worse with distance viewing Exodeviation worse with distance viewing Exodeviation worse with near viewing

Esodeviation worse with distance viewing A cranial nerve (CN) VI palsy, or a palsy of the abducens nerve, will cause an esodeviation on the affected side. This will result in horizontal diplopia that worsens with distance viewing because this nerve innervates the lateral rectus muscle. The patient may present with a head turn towards the same side as the affected eye. For example, if the patient has a right lateral rectus palsy, he or she may present with a head turn to the right to help eliminate diplopia. The lateral recti serve to abduct the eyes, and distance viewing requires divergence, or a turning out of both eyes simultaneously. A CN VI palsy will therefore be more evident when the patient looks far away, because the eye cannot abduct.

What is the MOST common type of oculomotor deviation observed in the general population? Hyperphoria Hypophoria Exophoria Esophoria

Exophoria By far the most common oculomotor deviations are exo in nature (approximately 95%), however most do not pose a problem. The least common type of deviation is vertical.

Which of the following classifications refers to an organism that can survive in an environment with or without oxygen? Obligate aerobe Facultative anaerobe Microaerophile Strict anaerobe

Facultative anaerobe Facultative anaerobes are capable of using oxygen as a life source but can also thrive using anaerobic respiration. When culturing this type of bacteria in a test tube of liquid media, one would observe the bacteria dispersed throughout the entire tube but with greatest concentration at the top, as aerobic respiration yields a more favorable energy result. Obligate aerobes require oxygen to survive. Test tube culturing would display bacteria gathering at the top of the tube where the oxygen levels are greatest. Microaerophiles are capable of utilizing oxygen, but only at lower concentrations. A test tube culture of microaerophillic bacteria would result in the greatest concentration of bacteria towards the upper portion of the test tube, but not at the very top. Strict anaerobes cannot tolerate oxygen and rely on anaerobic respiration to survive. A test tube culture of strict anaerobes would display the greatest concentration of bacteria towards the bottom of the tube where the least amount of oxygen is present. Aerotolerant organisms are capable of growing in the presence of oxygen but do not actually utilize it for respiration. One would observe these bacteria evenly distributed throughout a test tube liquid culture.

A positive catalase test indicates that a bacteria is capable of breaking down which of the following molecules? Glucose Pyruvate Hydrogen peroxide Carbon dioxide

Hydrogen peroxide Catalase is an enzyme commonly found in organisms that are exposed to oxygen. Catalase breaks down hydrogen peroxide into oxygen and water. The catalase test is performed by applying a drop of hydrogen peroxide to a microscope slide. A colony of bacteria is then exposed to the hydrogen peroxide via an applicator stick. The presence of bubbles or froth yields a positive catalase test. Staphylococci and Micrococci are catalase-positive organisms. Campylobacter and Escherichia coli are catalase-negative organisms.

Bipolar cells receive information from photoreceptors. Which of the following types of neurotransmitters do bipolar cells respond to? Glycine Dopamine Serotonin Glutamate

Glutamate Bipolar cells respond to glutamate released by photoreceptor cells. Glutamate release in the dark causes on-center bipolar cells to hyperpolarize (inhibition) and off-center bipolar cells to depolarize (excitation).

A 24-year-old female patient presents to your office complaining of side effects that began when she started using Patanol® to treat her ocular allergies. Which of the following symptoms is MOST likely associated with olopatadine (Patanol®) use? Visual hallucinations Depression Tachycardia Headache Gastrointestinal discomfort

Headache Topical mast cell stabilizer and antihistamine combination drops such as Patanol® (olopatadine) are commonly prescribed to relieve the symptoms associated with ocular allergies. These medications are typically very effective due to their dual action mechanisms. Topical allergy drops that possess this dual action are olopatadine (Patanol®), ketotifen fumarate (Zaditor®), azelastine (Optivar®), and epinastine (Elestat®). The aforementioned drops serve to alleviate itching and redness by blocking H1 receptors and inhibiting mast cell and basophil degranulation. Side effects of topical antihistamine/mast cell stabilizers include stinging upon instillation, headaches, and adverse taste (don't forget to inform your patients about punctal occlusion!). Tachycardia, depression, gastrointestinal discomfort, and visual hallucinations have not been reported with Patanol® use.

Which of the following microorganisms is associated with peptic ulcer formation?Campylobacter Clostridium tetani Vibrio cholerae Helicobacter pylori

Helicobacter pylori H. pylori is a helix-shaped, gram-negative, microaerophillic organism that colonizes in cells that secrete the mucosal lining of the stomach. These bacteria protect themselves from stomach acid by secreting urease, which, through a series of reactions, helps to neutralize the lower pH levels. H. pylori causes chronic gastritis, which in turn can lead to ulcer formation if pepsin and stomach acid levels increase to the point in which the protective mucosal lining is broken down. C. tetani is associated with the contraction of tetanus. V. cholerae causes cholera, resulting in abdominal cramps, diarrhea, fever, and vomiting.

An 81-year-old female reports that her eye has been watering more frequently over the past month. You decide to administer the primary Jones dye test (Jones I). After 5 minutes, the application of a cotton-tipped applicator to the inferior turbinate reveals the presence of dye in the area. Taking this into consideration, what is the MOST likely cause of the patient's epiphora complaint? Partial nasolacrimal duct obstruction Hypersecretion of tears Dysfunction of the valve of Hasner Punctal stenosis Complete nasolacrimal duct obstruction

Hypersecretion of tears The primary Jones dye test can be utilized to determine the patency of the nasolacrimal system. 1-2 drops of fluorescein are instilled into the inferior fornix of the eyes while the patient is in an upright position asked to blink her eyes normally. After a period of 5 to 10 minutes, a cotton-tipped applicator is used to swab the undersurface of the inferior turbinate on each side of the nasal passage. When the primary Jones dye test is positive (dye is recovered from the inferior turbinate of the nose), practitioners may conclude that the system is patent and that no significant blockage of the nasolacrimal drainage structure is likely. However, minor stenosis or physiologic dysfunctions cannot be completely ruled out. Patients who have a positive result on the Jones I test are more likely to experience symptoms of epiphora that are secondary to primary oversecretion of tears, rather than a dysfunction in lacrimal drainage (as in the above question). When the primary Jones dye test is negative, the probability of an obstruction or dysfunction in lacrimal drainage is much greater; however, this test alone is not sufficient to document this conclusion. The secondary Jones dye test is then necessary to determine the severity and location of the obstruction.

Purkinje images are caused by reflections of objects from the cornea and crystalline lens. Which of the four images moves forward as the eye accommodates? A. IV B. II C. I D. III

III There are four Purkinje images. The first image is caused by reflection from the anterior corneal surface and is the brightest of the images. The first image is roughly the same size as the object. The second Purkinje image is formed by the posterior surface of the cornea and almost coincides with the first Purkinje image. The third Purkinje image is the largest and is caused by reflection off of the anterior plane of the crystalline lens. The fourth Purkinje image is the smallest and is inverted, formed by reflection off of the posterior surface of the lens. During the process of accommodation the anterior surface of the lens moves forward. The image that is reflected off of this surface is Purkinje III. Therefore, Purkinje image III will move forward during accommodation.

A patient with chronic open angle glaucoma moves a 60-watt bulb on a flexible mounted arm from three feet to one foot from her newspaper. The illumination on the page will appear to have changed by how much? Increased by 3 times the original brightness Will be the same brightness Decreased by 1/9 of the original brightness Increased by 9 times the original brightness Decreased by 1/3 of the original brightness

Increased by 9 times the original brightness It has been said that prescribing low vision optical devices without consideration of the appropriate lighting will often doom the patient to failure. Unfortunately, there are no good tests to determine the exact type of lighting necessary for each patient's situation. Generally, different light levels are tried during the examination (as well as during the training session) with the patient using an adjustable light. The distance of a light source from the patient's reading material is very important because of the inverse-square law of illumination; the intensity varies inversely as the square of the distance from the page. If the light is moved from 1 foot to 3 feet from the page, a bulb will be needed that is approximately nine times as bright to keep the same illumination on the page. (It should be noted that technically, this relationship is only true for a point source of light.) Clinically, however, it gives a good approximation of the change in brightness (illumination) seen on the page when the distance of the light is changed. The illumination in the above example would therefore increase by 9X when the bulb is moved towards the page. Use the equation below to help solve this question: Illuminance = candle power of the source / (light source to surface distance)2 At 1 foot: illuminance = 60/(1)2 = 60/1= 60W At 3 feet: illuminance = 60/(3)2 = 60/9 = 6.7W Therefore, moving the light source from 3 feet (6.7W) to 1 foot (60W) will cause the brightness to appear to have increased by 9x (60/6.7 = 9).

A healthy retinal nerve fiber layer is thickest in which quadrant of the optic nerve head? Nasally Superiorly Inferiorly Temporally

Inferiority The nerve fiber layer is thickest at the inferior region, followed by the superior quadrant, then the nasal area, and is thinnest temporally. The inferior and superior arcades are composed of large diameter axons with little overlap of the receptive fields, thus explaining why a field defect usually occurs in these regions first for early cases of glaucoma. Inferior or superior notching of the nerve is highly suspect for glaucomatous damage, and patients with this finding must undergo further testing in order to rule out glaucoma. The next thickest area of nerve fiber layer tissue is nasally, which is comprised of the nasal radial fibers. These axons are affected in the later stages of glaucoma, thus explaining why a temporal island of the visual field is often left remaining in advanced cases of glaucoma. Lastly, the temporal rim area is usually the thinnest. Temporal rim tissue is comprised of the papillomacular bundle. The fibers in this area are very small and compact, with a high degree of receptive field overlap; therefore, because of the receptive field redundancy, a visual field defect correlating to this region will occur only after significant fiber loss has occurred. Due to the fact that these fibers are so small in diameter, even though they are numerous, the fibers do not occupy a lot of space in the optic nerve. The thickness of the nerve fiber layer rim tissue is best remembered as ISNT, with inferior being the thickest and temporal rim tissue being the thinnest.

A 12-year-old male accompanies his mother as she undergoes her annual eye exam. While waiting in the exam room, he eats a piece of candy that he found at your front desk. Unbeknown to him, the bar contains peanuts which he is severely allergic to. To counter potential anaphylactic shock, what would be the BEST course of action? Administration of oral prednisone Administration of Benadryl Use of topical olopatadine Injection of epinephrine

Injection of epinephrine Anaphylactic shock is defined as a severe, multi-system, type I hypersensitive, acute, allergic reaction that may be life-threatening. Signs of an allergic reaction include tingling, itching, hives, swelling of lips and tongue, constriction of the airway, vasodilation, myocardial depression, and a decrease in blood pressure. In these cases, the best option is use of an EpiPen (injected intramuscularly to the upper lateral thigh), as it provides immediate delivery of epinephrine to counter the reaction. Epinephrine (Adrenaline) activates both alpha and beta adrenergic receptors, causing an increase in peripheral vascular resistance, and allows for an increase in blood pressure and coronary artery perfusion. Adrenaline also serves to reverse vasodilation and decrease urticaria and angioedema. For severe, life-threatening reactions, Benadryl (diphenhydramine) will not work quickly enough. Topical antihistamines have little if any systemic absorption and therefore will not be effective in counteracting the anaphylaxis. While oral steroids may be useful in the post-management of anaphylactic shock, they will not yield the desired immediate response.

What is the relationship between the Abbe number and chromatic aberration? No relationship Equal Proportional Inversely proportional

Inversely proportional The Abbe number is the reciprocal of the dispersion of a spectacle lens. As the dispersion increases, a patient might experience seeing a rainbow of color as they view in the lens periphery. This is known as chromatic aberration. Crown glass has a high Abbe number of 59 and thus a low chromatic aberration.

A muscle that develops tension but does not shorten when activated displays which type of muscle contraction? Isovelocity Isometric Isovolume Isotonic

Isometric Isometric contraction occurs when a muscle is contracting but is not shortening. This type of muscle tension is used for load-bearing situations such as holding a plate of food in front of you. Muscles that shorten but maintain the same amount of tension are said to display isotonic contraction. An isovelocity contraction occurs when the force of the contraction varies while the velocity remains constant.

Which of the following skin conditions is considered to be benign and has the LOWEST risk of malignancy? Keratoacanthoma Actinic keratosis Squamous cell carcinoma Basal cell carcinoma

Keratoacanthoma Keratoacanthoma appears very much like a squamous cell carcinoma (SCC) in that the lesion tends to progress rapidly and ulcerates. This condition typically occurs in middle-aged and elderly patients of Caucasian descent on areas of the skin that are exposed. The lesion appears elevated and the center will eventually produce a scab-like plug of keratin. The margins surrounding the plug will be rolled. At some point the keratin plug will fall out, resulting in the formation of a pit, then the lesion will regress. Most clinicians will have this type of lesion biopsied due to its similarities to SCC. Actinic keratosis is a pre-cursor to squamous cell carcinoma and appears as scaly, dry skin that does not heal. People with skin that is of lighter pigmentation, and those with excessive exposure to ultraviolet light tend to be most at risk for development of this condition. Squamous cell carcinoma (SSC) is thankfully one of the rarest malignancies, but due to its ability to metastasize, it can be quite dangerous. This malignancy has the ability to progress rapidly and has a high affinity for people who spend a lot of time in the sun, especially those who are light-skinned. The only way to definitively diagnose SCC is to refer for a biopsy. Removal of these lesions should occur using Mohs technique. Essentially, Mohs procedure calls for continued removal of tissue and biopsy of the surrounding borders. If the borders prove to be malignant then more tissue is removed and biopsied. This continues until the borders prove to be free of any carcinoma. Basal cell carcinoma (BCC) is the most common malignant lid lesion and mercifully tends to be very slow-growing. BCC generally appears as a waxy, translucent nodule. Eventually the nodule will ulcerate. Patients may bring these to your attention and tell you that they have "had it for years and it just does not seem to heal". Whenever you hear this it is best to send out for biopsy. BCC very rarely metastasizes.

Which of the following is the correct pathway for the drainage of tears through the nasolacrimal drainage system? Lacrimal sac → lacrimal punctum → lacrimal canaliculus → nasolacrimal duct → valve of Hasner Nasolacrimal duct → lacrimal sac → valve of Hasner → lacrimal canaliculus → lacrimal punctum Lacrimal punctum → lacrimal canaliculus → lacrimal sac → nasolacrimal duct → valve of Hasner Lacrimal punctum → lacrimal canaliculus → valve of Hasner → nasolacrimal duct → lacrimal sac

Lacrimal punctum → lacrimal canaliculus → lacrimal sac → nasolacrimal duct → valve of Hasner The lacrimal punctum is a small aperture located in the lacrimal papilla; initially, the tear film drains through this aperture. The lacrimal punctum leads into the lacrimal canaliculus, the tube connecting the punctum to the lacrimal sac. The ampulla is a slight dilation in the initial portion of the lacrimal canaliculus. The canaliculi from the upper and lower lids run horizontally along the lid margin, connecting into a common canaliculus that then enters the lateral aspect of the lacrimal sac located in the anterior portion of the medial orbital wall. The lacrimal sac empties into the nasolacrimal duct in the maxillary bone. The valve of Hasner is located at the terminus of the nasolacrimal duct in the inferior nasal meatus. The Valve of Hasner is a fold of mucosal tissue that ensures that fluid flows anterograde out of the duct.

Which of the following muscle pairings and actions follows Sherrington's law of reciprocal innervation? Looking to the right causes contraction of the right medial rectus and inhibition of the right lateral rectus Looking to the right causes contraction of the right lateral rectus and inhibition of the right medial rectus Looking to the right causes contraction of the right medial rectus and contraction of the left medial rectus Looking to the right causes contraction of the right lateral rectus and inhibition of the left medial rectus

Looking to the right causes contraction of the right lateral rectus and inhibition of the right medial rectus Sherrington's law of reciprocal innervation states that when a muscle is stimulated to contract, its antagonist is inhibited. Based upon this law, looking to the right causes contraction of the right lateral rectus and inhibition of the right medial rectus.

Numerous reports have suggested that increased tear film osmolarity is a key factor in dry eye. Tear osmolarity increases in most dry eye sub-types due to which of the following processes? Reactive oxygen species are increased in the tears of most dry eye sub-types; this increases osmolarity The lipid layer is altered in most dry eye states, leading to ion pairing The lacrimal gland produces more ionic species Decreased capillary exchange leads to ionic bonding Patients with dry eye tend to blink less than normals, leading to increased evaporation Loss of tear stability induces an increased evaporation rate, leading to increased osmolarity

Loss of tear stability induces an increased evaporation rate, leading to increased osmolarity Tear film instability increases osmolarity through of a mechanism of concentrating the tears on the eyes; evaporation of the aqueous component of the tears occurs, leaving the ionic species in a lesser amount of tears. This increase the pH of the tears, thereby also increasing the osmolarity. Several studies have indicated that normal tear osmolarity is less than or equal to 300 Osm/L, with values exceeding 308 Osm/L indicating increased osmolarity. As a single measure, tear osmolarity has recently been found to correlate the best with the severity of dry eye when compared to several other clinical tests in a large, multi-center study (Sullivan et al., IOVS 51:6125-6130, 2010).

Chronic blepharitis, if left untreated, can cause which of the following structural changes to the anterior segment of the eye? Madarosis Tristichiasis Hypertelorism Distichiasis

Madarosis Blepharitis is a condition caused by pathogens, usually of Staphylococcus origin, that colonize along the eyelid margins. The bacteria produce exotoxins which take the form of flakes and are generally seen along the base of the eyelashes. Unfortunately, this condition is chronic but will wax and wane in its presentation. Long-term complications include madarosis (missing lashes), trichiasis, neovascularization of the eyelid margin, keratitis, erythema, phlyctenule formation and infiltrates. Patients may complain of dry, irritated eyes, stinging, pain, itching, frequent eye infections, foreign body sensation, and decreased acuity (if there is corneal involvement). Treatment includes eyelid scrubs, antibiotic ointments, and in some cases transient topical steroid use to decrease lid inflammation (usually used in conjunction with a topical antibiotic). Occasionally oral antibiotics are prescribed, especially in the event of poor compliance. Distichiasis is a rare phenomenon (either congenital or acquired) that is marked by abnormal growth of eyelashes from the orifices of the meibomian glands. Hypertelorism is a term used to describe the incidence in which the orbits are located quite far apart. This generally occurs along with other congenital cranium anomalies. Tristichiasis is a very rare occurrence in which a person possesses three rows of eyelashes.

Midget ganglion cells receive pre-synaptic information from which of the following cells? Horizontal bipolar cells Midget bipolar cells Rod bipolar cells Flat bipolar cells

Midget bipolar cells Midget bipolar cells synapse onto midget ganglion cells. These are very selective and exclusive channels as one cone cell synapses with one midget bipolar cell, which then relays the information to one midget ganglion cell. There is no additional input from other cells or synapses. These types of monosynaptic cells are most common in the central retina, thus explaining the ability to visually discern fine details. Flat bipolar cells receive information from many cone cells and in turn synapse with many ganglion cells. Rod bipolar cells, as their name suggests, convey information from many rod cells to several ganglion cells. Rods relay information only to rod bipolar cells.

A patient is concerned with an acute reduction of visual acuity in his right eye. You correctly diagnose central serous retinopathy, and confirm your diagnosis with optical coherence tomography (OCT). What is the standard treatment protocol for this condition? Refer for laser treatment of the retina Refer for intravitreal steroid injection Refer for cryotherapy of the retina Monitor monthly for resolution Treat the patient with prism as he is likely to develop diplopia

Monitor monthly for resolution CSR is more commonly seen in middle-aged males under high-stress, who are very anxious, or have type A personalities. This condition causes fluid to leak from the choriocapillaries into the subretinal area, causing a serous detachment of the neurosensory retina. There is usually an associated loss of the foveal reflex, a hyperopic shift, a potential relative scotoma, and metamorphopsia. Flourescein angiography will reveal hyperfluorescence that appears like a smoke-stack. Evaluation of the posterior pole will typically display a blister-like elevation of the neurosensory retina. Patients with this condition are typically monitored monthly and intervention is rarely required, as most cases of CSR will resolve within roughly 6 months.

Which genus of organisms is responsible for tuberculosis and leprosy? Borrelia Salmonella Mycobacterium Klebsiella

Mycobacterium Mycobacterium is an aerobic, Gram-positive (although this is somewhat debatable) genus that includes several pathogenic species. Mycobacteria are extremely difficult to treat due to the nature of their cell walls which are truly neither Gram-negative nor Gram-positive (although they are classified as Gram-positive because they are acid-fast). M. tuberculosis is responsible for causing tuberculosis. M. leprae is the culprit that causes Hansen's disease (also termed Leprosy). A species of Borrelia can cause Lyme disease. Salmonella has been known to cause food poisoning. Contraction of Klebsiella can lead to the development of pneumonia.

What is the MOST common side effect associated with intravenous administration of sodium fluorescein in patients requiring fluorescein angiography? Bronchospasm Localized tissue necrosis Nausea and vomiting Anaphylaxis Elevated temperature

Nausea and vomiting The most common reaction observed in patients undergoing intravenous injection of sodium fluorescein is transient nausea and occasional vomiting, which typically occurs within 30-60 seconds after administration in fewer than 5% of patients. Moderate adverse side effects can occur in up to 1% of patients, which include thrombophlebitis, localized tissue necrosis, nerve palsies, and elevated temperature. Severe and potentially life-threatening complications can result in laryngeal edema, bronchospasm, circulatory shock, myocardial infarction, and anaphylaxis; however, these adverse reactions occur in an extremely low percentage of patients.

A cortical hypercolumn is comprised of which of the following? An ocular dominance column for one eye only and orientation columns for one specific orientation only Ocular dominance columns for both eyes and orientation columns for one specific orientation only A complete set of ocular dominance columns for one eye only and a complete set of orientation columns An ocular dominance column for one eye only and a complete set of orientation columns Ocular dominance columns for both eyes and a complete set of orientation columns

Ocular dominance columns for both eyes and a complete set of orientation columns The striate cortex is organized into discrete rows and columns that help to code for specifics of a stimulus. A hypercolumn consists of both a right and left eye ocular dominance column, as well as orientation columns for every orientation. An electrode that penetrates the cortex perpendicularly will encounter cells with the same ocular dominance, and they all respond to stimuli of the same orientation. However, an electrode that penetrates the cortex parallel to its surface will encounter neurons that all possess the same ocular dominance but respond preferentially to stimuli of different orientations. In order for ocular dominance columns to form properly, it is essential that normal vision is present in both eyes during the early years of life.

Which of the following is the correct order of structures through which pupillary fibers course as they travel through the afferent pupillary pathway? Optic nerve → optic chiasm → brachium of the superior colliculus → pretectal region of the midbrain → Edinger-Westphal nucleus Optic nerve → optic chiasm → Lateral geniculate nucleus in the thalamus. → Edinger-Westphal nucleus Optic nerve → optic chiasm → optic tract → pretectal region of the midbrain → Lateral geniculate nucleus in the thalamus Optic nerve → optic chiasm → optic tract → Lateral geniculate nucleus in the thalamus

Optic nerve → optic chiasm → brachium of the superior colliculus → pretectal region of the midbrain → Edinger-Westphal nucleus The afferent pupillary fibers exit the eye through the optic nerve and pass through the optic chiasm, where they then exit the optic tract and enter into the brachium of the superior colliculus and synapse onto cells in the pretectal area of the midbrain. The pathway then continues and stimulates intercalated neurons, which in turn stimulate cells in the Edinger-Westphal nucleus. The axons of the pupillary pathway never enter the lateral geniculate nucleus.

Long-term use of corticosteroids can lead to the formation of which of the following types of cataract? A. Anterior subcapsular cataract B. Nuclear sclerotic cataract C. Cortical cataract D. Posterior subcapsular cataract

Posterior subcapsular cataract The possible formation of posterior subcapsular cataracts (PSC) is a common concern in patients undergoing long-term treatment with corticosteroid therapy. PSCs have been associated with the use of systemic, topical, ophthalmic, topical dermatologic, nasal aerosol, and inhalation type steroids. This relationship is likely dose-dependent, and the average time from beginning steroid treatment to the onset of lens changes is approximately 1 year (with a dosage of 10 mg/day of prednisone), but lens changes have been observed in as short as 2 months with as little as 5 mg/day. Patients with PSC formation may complain of an increase in light sensitivity, photophobia, glare, or difficulty reading. If visual acuity is notably decreased, surgical removal of the lens may be warranted.

A 31-year-old male patient presents to your office for a photorefractive keratectomy (PRK) pre-operative examination. As you review his required ocular medication schedule, which of the following prescribed drops must you remember to tell him to "shake well" before instillation? FreshKote® Acular® Zymaxid® Pred Forte®

Pred Forte® - an ocular medication that is in suspension form; therefore, it is important to shake this medication well before use. Other forms of prednisolone that are suspensions include Pred Mild®, Econopred®, and Econopred Plus®. On the other hand, there are prednisolone ocular medications that are solutions, making shaking of the bottle unnecessary. These include AK-Pred®, Inflamase Mild®, and Inflamase Forte®. Zymaxid® is a 4th generation fluoroquinolone antibiotic that is bottled in solution form, as is Acular®, which is an ocular non-steroidal anti-inflammatory drug (NSAID). FreshKote® is an artificial tear that also does not need to be shaken before use.

A patient is seen at your office complaining that her right eye appears higher than her left eye. She would like to know if glasses would help improve the cosmesis of her asymmetry. You know that prism will shift the image of an object. How would you orient a prism to help her appearance? Prescribe base down prism over the right eye Prescribe base in prism over the right eye Prescribe base up prism over the right eye Prescribe base out prism over the right eye

Prescribe base up prism over the right eye A prism will bend light towards its base, but the image will be shifted towards the apex of the prism. Therefore, by prescribing base up prism over her right eye, its image will be shifted down towards the apex of the prism. Another way of remembering this is to think of the prism as an arrow that will point in the direction of the deviation (i.e., exotropia is neutralized with base in prism, the eye points outwards, the apex of the prism also points out). Prescribing prism for cosmetic purposes may not always be an option, as significant vertical prism may induce diplopia or visual discomfort.

Which type of light scattering is responsible for the reddish-orange colors that are often observed during sunsets? Mie scattering Rayleigh scattering Brillouin scattering Tyndall scattering Raman scattering

Rayleigh scattering Scattering of light occurs when the medium through which light or other electromagnetic radiation travels is not homogenous. In the case of Rayleigh scattering, the particles that scatter the light are smaller than the wavelength of the light passing through. The particles may be individual atoms or molecules of a solid, liquid, or most commonly, a gas. The appearance of the blue sky during the day and the reddish hue of the sunset are due to Rayleigh scattering of light.

In addition to flattening the base curve, which of the following alterations will help to loosen a tightly-fitting gas permeable (GP) contact lens? Increase the overall diameter Reduce the width of the peripheral curve system Steepen the peripheral curve system Decrease the central thickness Reduce the size of the optic zone

Reduce the size of the optic zone There are a multitude of alterations that can be made when a GP contact lens is fitting too tightly, many of which can be done in-office if a modification unit is available. If a gas permeable lens is fit too tightly, flattening of the base curve is the most common alteration. One can also decrease the optic zone, decrease the overall diameter (OAD), widen the peripheral curve system, or flatten the peripheral curve system. In order to modify a lens that is fitting too loosely, simply reverse all of the above: steepen the base curve, increase the OAD, increase the optic zone, steepen the peripheral curve system, and narrow the width of the peripheral curves.

One of your tech-savvy low vision patients wishes to use a closed-circuit television (CCTV) for reading. A CCTV operates on which of the following principles of magnification? Rated magnification Relative size magnification Relative distance magnification Equivalent magnification

Relative size magnification A CCTV works on the principle of relative size magnification (or projection). It operates by enlarging text on the screen without the need for any lenses in front of the patient, or the patient moving closer to the device. When the print is enlarged electronically in this matter, the image of the print subtends a larger area on the retina and thus appear larger in size. An example of relative distance magnification would be if you were holding a newspaper at 40 cm and you moved it closer to 20 cm. The print now appears 2 times as large relative to the 40 cm distance. Rated magnification is often used by manufacturers of some hand magnifiers and stand magnifiers using a 25 cm reference distance.

Which of the following bifocal lens designs will create the largest amount of image jump? Executive bifocal Round 28 bifocal FT 28 bifocal FT 35 bifocal

Round 28 bifocal Image jump occurs when a patient's eyes move from viewing through the distance carrier of a bifocal lens into the lens segment. As the eyes enter the near segment, at first, they are not aligned with the near optical center (OC); this induces a prismatic effect. The farther away the near OC is from the top of the segment, the larger the amount of image jump that will occur. The round 28 bifocal has its optical center 14 mm below the top of the segment (the optical center for a round segment is located at its center, which is half of its diameter). This large distance creates significant image jump. The flat top 28 and flat top 35 bifocal optical centers are located 5 mm below the segment line. The Executive OC is on the segment line, so no image jump is created.

What type of agar is commonly used to culture fungi? Cetrimide agar Blood agar Sabouraud's agar Thayer-Martin agar Hay infusion agar

Sabouraud's agar Sabouraud's agar is useful for culturing fungi. Culturing and sensitivity testing are important diagnostic tools used to determine the offending pathogen and help to select the appropriate medication for treatment. Sabouraud's agar is unique in that it possesses a low pH, causing inhibition of most bacterial growth (allowing for better isolation of the fungus). Hay infusion agar is frequently utilized to culture slime moulds. Cetrimide agar is designed to isolate Gram-negative bacteria. Blood agar plates are useful for the detection of pathogenic organisms via the presence of hemolytic activity (the ability of organisms to lyse or destroy red blood cells). Blood agar plates are not a selective medium, as many different types of organisms are capable of growth on this type of agar. Thayer-Martin agar is a type of chocolate agar used to isolate Neisseria gonorrhoeae.

Which 2 of the following types of scleritis, although extremely rare, can lead to scleral perforation? (Select 2) Nodular scleritis Diffuse scleritis Scleromalacia perforans Necrotizing scleritis

Scleromalacia perforans Necrotizing scleritis Scleritis is an inflammation of the sclera that generally occurs secondarily to a systemic condition, usually of collagen vascular origin. Diffuse scleritis has a gradual onset and presents as boring eye pain that may radiate to other structures such as the jaw and forehead. Patients will present with distension of the scleral vascular pattern, causing a deep pinkish hue of the sclera. Nodular scleritis appears similar to diffuse scleritis, but the areas of inflammation are localized to painful, raised nodules. Scleromalacia perforans is the least common form and is almost always seen in association with rheumatoid arthritis. Patients with scleromalacia perforans generally do not experience pain or inflammation. Necrotizing scleritis is the most severe form and has a higher mortality rate than the other types due to the fact that it usually stems from severe autoimmune disease

What is the name of the pigmented line that is located at the leading edge of a pterygium? Stocker line Hudson-Stahli line Coat white ring Ferry line Fleischer ring Krukenberg spindle

Stocker line - Stocker line is a deposition of iron in the corneal epithelium that is located at the leading edge of a pterygium - A Hudson-Stahli line is an iron line that is commonly observed at the junction of the middle and lower third of the cornea (where lid closure occurs upon blinking) - Ferry line is found anterior to a filtering bleb - Coat white ring is a small, white, oval ring at the level of Bowman's membrane that is associated with a previous corneal foreign body - A Fleischer ring is an iron pigment ring that encircles the base of a cone in keratoconus - A Krukenberg spindle is a deposition of pigment on the corneal endothelium that is associated with pigment dispersion syndrome

An eyelid ptosis can be caused by dysfunction or damage to which of the following muscles? Superior tarsal muscle (muscle of Muller) Muscle of Horner Inferior rectus Pars ciliaris (Riolan's muscle)

Superior tarsal muscle (muscle of muller) Ptosis is a condition in which the upper eyelid sags. It can be caused by dysfunction of either the superior palpebral levator or the superior tarsal muscle (muscle of Muller). Because the levator is the major muscle responsible for raising the upper eyelid, ptosis from levator damage is often more severe than ptosis from dysfunction of the muscle of Muller. The muscle of Horner (also known as the pars lacrimalis) is part of the palpebral portion of the orbicularis oculi. The fibers for the muscle of Horner come from the lacrimal crest and encircle the lacrimal canaliculi. This assists the flow of tears into the nasolacrimal drainage system when the orbicularis oculi contracts to close the eye. The muscle of Riolan (also known as the pars ciliaris) is another section of the palpebral portion of the orbicularis oculi; it lies near the lid margin to maintain eyelid apposition to the globe. The orbicularis oculi is the major muscle responsible for closing the eyelids.

According to the Keith-Wagener-Barker method of classification, hypertensive retinopathy is categorized as stage 4 when which ocular sign is present? Flame hemorrhages Retinal edema Hard exudates in a star configuration Swelling of the optic disc

Swelling of the optic disc Grading of hypertensive retinopathy according to the Keith-Wagener-Barker system is as follows: Stage 1- narrowing of the retinal arteries Stage 2- stage 1, plus focal constriction of the retinal vasculature (arteriovenous nicking) Stage 3- stage 2, plus retinal hemorrhages, hard exudates (likely in a star configuration), cotton wool spots, and retinal edema Stage 4- stage 3, plus swelling of the optic disc. Patients with this stage must be hospitalized immediately.

The ligaments that suspend the crystalline lens (zonules) are embryonically derived from what structure? The tertiary vitreous The primary vitreous The lens capsule The lens epithelium

Tertiary vitreous The zonules are attached to the posterior and anterior surfaces of the lens and connect to the pars plana of the ciliary body. The primary vitreous develops from gestational weeks 3 through 9. The secondary vitreous then begins to form and condenses the primary vitreous, forming Cloquet's canal. Developmentally, the tertiary vitreous is secreted last; the zonules are comprised of condensed tertiary vitreous.

Which of the following medications should be taken with an empty stomach? Amoxicillin with clavulanate (Augmentin®) Tetracycline Cephalexin (Keflex®) Doxycycline

Tetracycline Tetracycline has a half-life of 8 hours and is classified as a short-acting drug. It is most effective when taken on an empty stomach due to the fact that calcium, iron, and magnesium can decrease the effectiveness or inactivate this medication. Remember, tetracycline should NEVER be prescribed to children because it will deposit in developing teeth and bones. Doxycycline has a half-life of 18 hours and is therefore categorized as a long-acting tetracycline. Doxycycline can be taken with or without food. Although it is more costly, doxycycline is generally favored over tetracycline because it causes less gastrointestinal upset, therapeutic benefits can be achieved with smaller doses, less phototoxicity is experienced by the patient, and there is a decreased risk of kidney and liver toxicity. The effectiveness of Augmentin® and Keflex® remain unaltered if food is ingested.

During gestation, when does the secondary vitreous begin to develop? The 1st week of gestation The 9th week of gestation The 20th week of gestation The 30th week of gestation

The 9th week of gestation The primary vitreous develops at around the third week of gestation. It is formed by mesoderm. The secondary vitreous begins to develop during the ninth embryonic week and later becomes the mature vitreous. The secondary vitreous stems from primary vitreal cells and retinal glial cells and therefore originates from neuroectoderm. The secondary vitreous expands to fill the globe while compacting the primary vitreous into the center of the globe.

What is the name for the phenomenon in which a flickering light that is 10 Hz is seen as brighter than a steady light (one that does not flicker) with the same average luminance? The Purkinje tree The Brucke-Bartley effect The Granit-Harper law The Troxler effect

The Brucke-Bartley effect The Brucke-Bartley effect describes the fact that a flickering stimulus that is 10 Hz will appear brighter than a non-flickering light with the same average luminance. This fact also holds true for stimulus presentation duration. A light that is presented for 50 milliseconds will appear brighter than stimuli presented for longer or shorter durations. The Troxler effect occurs when the eye is fixated (although the eye is truly always moving) on a point in space and the surrounding background begins to blend together. There must be several factors that come into play in order for the Troxler effect to transpire. The best example of this phenomenon is the figure in which there are two squares that are superimposed. The smaller square is centered in the larger square and is slightly lighter than the larger surrounding square. The border between the squares is blurred, resulting in a distinction of the two squares based upon brightness alone. When fixating upon an X placed in the center of the smaller square, the border completely disappears as does the smaller square, resulting in the perception of one uniformly-colored large square. Some patients experience the Troxler effect while performing the FDT visual field and they report, especially during testing of the second eye, that the entire field appears to go gray. When this occurs make sure to inform the patient to blink. The peripheral retina is most sensitive to flicker because it is a part of the magno system which is also known as the "where" system. The magno system is known for its poor spatial resolution but good temporal resolution. The fact that the peripheral retina is more sensitive to flicker is in accordance with the Granit-Harper law, which states that as the log of the area of the stimulus is increased, the critical flicker fusion frequency also increases accordingly. This helps to explain why the chances of perceiving flicker are greater for a larger stimulus for a given modulation. Remember that the receptors of the peripheral retina display increased summation; therefore, a larger stimulus will take up more area of the retina, increasing the chances of detection. Due to the fact that the eye does not respond well to low temporal frequencies, stabilized retinal images are unable to be detected. Blood vessels that lie on top of the photoreceptors are considered stable relative to the retina. However, if you shine a penlight against your eyelid and move it rapidly, you will be able to see the shadows cast by the blood vessels on your retina. This is called a Purkinje tree. The penlight must be moving in order to ensure that the temporal frequency is great enough to be visualized.

A central retinal artery occlusion (CRAO) causes tremendous damage to the retina. How will the electroretinogram (ERG) of a person who has suffered a CRAO be affected? Both the a-wave and the b-wave will disappear The a-wave will remain but the b-wave will disappear The a-wave will disappear but the b-wave will remain Both the a-wave and the b-wave will remain

The a-wave will remain but the b-wave will disappear The b-wave is formed by responses from the bipolar and Muller cells, both of which are nourished by the central retinal artery. The a-wave results from excitation of the photoreceptors. The a-wave will not be lost in the event of a CRAO due to the fact that photoreceptors receive their oxygen supply via the choroid.

Dacryoadenitis refers to inflammation or infection of which of the following ocular structures? The puncta The nasolacrimal sac The lacrimal sac The lacrimal gland

The lacrimal gland Dacryoadenitis describes inflammation of the lacrimal gland, generally due to infection. The swelling can be categorized as either chronic or acute. Acute presentations appear more commonly as a unilateral swelling of the upper eyelid, along with pain, excessive lacrimation, probable ipsilateral lymphadenopathy, and potential proptosis. If the condition is bilateral, it is likely due to a systemic infection. Chronic dacryoadenitis is generally bilateral and presents with hard masses that are palpable at the locations of the lacrimal glands. This form is often painless and caused by inflammatory diseases such as Grave's, Sjogren's, or sarcoidosis. Chronic dacryoadenitis often warrants further investigation in order to rule out a lacrimal gland tumor.

Which extraocular rectus muscle has its insertion site CLOSEST to the limbus? The lateral rectus The superior rectus The medial rectus The inferior rectus

The medial rectus The medial rectus inserts into the sclera roughly 5.3 mm from the limbus, followed by the inferior rectus, which inserts 6.8 mm from the limbus. The lateral rectus inserts 6.9 mm from the limbus, and the superior rectus has the furthest insertion point at 7.9 mm from the limbus. Remember MILS (Medial rectus, Inferior rectus, Lateral rectus, Superior rectus). If one draws a line connecting the insertion points of the muscles, an imaginary spiral is created called the spiral of Tillaux.

Which of the following may occur if you increase the water content of a soft (non-silicone) hydrogel contact lens? The patient may report an increase in dry eye symptoms The oxygen permeability may decrease The lens durability may increase The tendency of lens deposits may decrease

The patient may report an increase in dry eye symptoms Typically, as the water content of a soft (non-silicone) hydrogel contact lens increases, the durability of the lens will decrease, the oxygen permeability of the lens will increase, and deposit formation and dry eye symptoms will also increase. This is also mostly true for silicone hydrogel lenses, except that increasing the water content of the lens may actually decrease the oxygen permeability.

Berger's space is created by an interval between which two structures? A. The equator of the lens and the ciliary body B. The anterior face of the lens and the posterior surface of the iris C. The posterior surface of the cornea and the anterior face of the iris D. The posterior face of the lens and the anterior vitreous

The posterior face of the lens and the anterior vitreous Berger's space is created by the separation between the posterior face of the lens and the anterior face of the vitreous. The space between the equator of the lens and the ciliary body is known as the circumlental space.

The lymphatic system serves many important roles in the human body. The lateral portion of the eyelid lymphatics drain into which of the following structures? The conjunctiva The pre-auricular lymph node The puncta The submandibular lymph node

The pre-auricular lymph node The lateral 2/3 of the upper lid and the lateral 1/3 of the lower lid lymphatics drain into the pre-auricular lymph node located directly in front of the ear. The medial 1/3 of the upper eyelid and the medial 2/3 of the lower lid lymphatics drain into the submandibular node located just under the jaw-line. Therefore, it is very important to evaluate these two nodes separately, especially when a condition of viral etiology is suspected.

Which of the following BEST describes the definition of irregular astigmatism? The principal meridians of the cornea are located 90 degrees apart The axis of astigmatism is located along an oblique axis The axis of astigmatism is located along the 90 degree meridian The principal meridians of the cornea are not perpendicular to each other

The principal meridians of the cornea are not perpendicular to each other Astigmatism can be classified as either being regular or irregular. Regular astigmatism occurs in individuals in which the principal meridians of the cornea are located 90 degrees apart. That is, the area of the cornea with the flattest curvature (the axis) is oriented perpendicular to the meridian of the steepest curvature. In certain ocular conditions such as keratoconus, corneal scarring, or post-surgical corneas, the steep and flat meridians may not be oriented 90 degrees apart. This type of corneal curvature can be considered irregular astigmatism. In these cases, the refractive error is typically not well corrected with spectacles and is often better corrected with gas-permeable contact lenses.

Drusen typically deposit between which of the following layers of the retina? The retinal pigment epithelium and Bruch's membrane The ganglion cell layer and the nerve fiber layer The inner and outer nuclear layers The inner and outer plexiform layers

The retinal pigment epithelium and Bruch's membrane Drusen deposits collect between the retinal pigment epithelium (RPE) and Bruch's membrane. The retinal pigment epithelium plays a very important role in phagocytosis of shed outer segments of photoreceptors. If the RPE fails to rid the retina of this debris, it will begin to accumulate, which can have a significant impact on vision and may lead to macular degeneration.

Which layer of the cornea, if penetrated, will leave a scar? The wing cell layer The stroma The superficial cell layer The basal cell layer

The stroma The corneal epithelium is comprised of 3 major layers. The outermost layer is composed of superficial cells (2-3 layers) followed by wing cells (2-3 layers) and lastly, basal cells (1 layer). Damage to any of the layers of the corneal epithelium will heal without scar formation. The epithelial basement membrane (Bowman's layer) is made up of collagen types IV, VII and XII. The stroma makes up the bulk of the cornea and is comprised of keratocytes, nerves, type I collagen fibers and mucopolysaccharides. If injured, the stroma will heal but a scar will most often remain at the site of trauma.

You are measuring the palpebral fissure height in a patient who reports drooping of his upper eyelid. Which of the following BEST describes the normal positioning of the upper and lower eyelids in comparison to the limbus? The upper lid normally rests about 1mm below the upper limbus, and the lower lid rests about 2mm above the lower limbus The upper lid normally rests about 2mm below the upper limbus, and the lower lid rests about 1mm above the lower limbus The upper lid normally rests about 2mm below the upper limbus, and the lower lid rests about 1mm below the lower limbus The upper lid normally rests about 1mm below the upper limbus, and the lower lid rests about 2mm below the lower limbus

The upper lid normally rests about 2mm below the upper limbus, and the lower lid rests about 1mm above the lower limbus The palpebral fissure height is a measurement of the distance between the upper and lower eyelid margins when the patient is looking in primary gaze. This particular measurement is typically less in males (7-10mm) as compared to females (8-12mm). The normal positioning of the upper and lower eyelids are as follows: the upper eyelid usually rests about 2mm below the superior limbus, while the lower eyelid position is typically 1mm above the lower limbus. A unilateral ptosis can be quantified by comparing these measurements to the contralateral eye. A ptosis of up to 2mm may be graded as mild; a 3mm ptosis is considered moderate; a ptosis of 4mm or more is deemed severe. Another important measurement in evaluating a ptosis is the marginal-reflex distance (MRD). The MRD can be defined as the distance between the upper eyelid margin and the resultant corneal reflection caused by directing a patient's gaze at a penlight held by the examiner. This measurement is normally 4-4.5mm.

A common cause of epiphora in infants is due to a small membrane that covers over which of the following structures? The canaliculus The lacrimal gland The puncta The valve of Hasner

The valve of Hasner It is common for parents of young infants to note that one eye (or both eyes) of their child constantly tears and has mucopurulent discharge. This epiphora results from a blockage of the nasolacrimal passageway caused by a membrane covering the valve of Hasner. The majority of blockages will self-resolve without intervention (80-90% of infants) within the first 12 months of life. However, some clinicians may recommend massage of the nasolacrimal sac several times a day in an effort to rupture the membrane.

Which of the following terms describes a constant ringing of the ears? Otitis media Tinnitus Malleus Sinusitis

Tinnitus is caused by damage to hair cells in the inner ear from exposure to excessive noise, medications (like aspirin), aging, and some systemic diseases. Sound waves cause the hair cells to bend, releasing a neurotransmitter that leads to action potentials of the auditory nerve. Sometimes the hair cells break or are left in the "on" position, causing the perception of ringing. Otitis media is an infection or inflammation of the middle ear. Sinusitis refers to inflammation of the sinuses. The malleus, also known as the hammer, is one of the tiny bones in the ear that helps to transmit and amplify sound to the auditory nerve.

While examining a patient's right eye with diplopia, you ask him to look downward and toward his nose. He is able to move the eye toward his nose (medially) but not down. Based on the isolated agonist model of eye movement by extraocular muscles, which nerve and muscle are not functioning appropriately? Oculomotor nerve, superior rectus Abducens nerve, inferior oblique Trochlear nerve, inferior oblique Trochlear nerve, superior oblique Abducens nerve, lateral rectus

Trochlear nerve, superior oblique The trochlear nerve (CN IV) innervates the superior oblique muscle. The abducens nerve (CN VI) innervates the lateral rectus, which is not involved in the motion described in this question. The oculomotor nerve has two divisions; the inferior division innervates the inferior rectus, inferior oblique and medial rectus, while the superior division innervates the superior rectus and levator palpebrae superioris. The functions and anatomy of the extraocular muscles are as follows: Superior rectus - turns eye up, adducts, and medially rotates (intorsion) Inferior rectus - turns eye down, adducts, and laterally rotates (extorsion) Lateral rectus - abducts eye (laterally) Medial rectus - adducts eye (medially) Superior oblique - medially rotates (intorsion), abducts and turns eye down Inferior oblique - laterally rotates (extorsion), abducts and turns eye up In the case described here, when the patient adducts the eye medially with the medial rectus as well as the superior and inferior rectus, only the superior oblique and inferior oblique can move the eye down or up respectively because the superior and inferior rectus muscles are already contracted. The same is true if a patient abducts the eye with the obliques and lateral rectus: only the superior and inferior rectus can move the eye up or down respectively

A patient is seen at your office reporting constant diplopia. The patient notes that the diplopia is still present when you cover her right eye. Based upon this information, what is the MOST likely etiology of her diplopia? Lateral rectus palsy Aneurysm Uncorrected refractive error Superior oblique palsy

Uncorrected refractive error Monocular diplopia is never caused by any type of cranial nerve dysfunction. The most common cause of monocular diplopia is uncorrected refractive error. Other causes of monocular diplopia include corneal irregularities, lens irregularities (cataract), lens subluxation (very rare), or an improper glasses prescription. Whenever you are confronted with a recent onset of diplopia, the first thing you must determine is whether the diplopia is present monocularly or binocularly.

Which area of the extrastriate cortex is involved in the perception of motion? The inferotemporal cortex (IT) Visual area 1 (V1) Visual area 4 (V4) Visual area 5 (V5) Visual area 2 (V2)

Visual area 5 (V5) Beyond the striate cortex, visual information is thought to split into two separate streams: the ventral (also known as the "what" or temporal stream), and the dorsal (also known as the "where" or parietal stream). V5, also known as the middle temporal cortex (or MT), is considered to be a part of the dorsal stream and is believed to code for motion. V4 and IT are a part of the ventral stream. V4 serves to play a role in processing color, while IT is important in classifying complex shapes and form recognition (such as faces). V1 is the primary visual cortex and is not considered a part of the extrastriate cortex.

Which of the following methods can be used to test for the presence of eccentric fixation? The Hirschberg test The Bruckner test Binocular versions Visuoscopy

Visuoscopy Angle Kappa (Lambda), visuoscopy, Haidinger Brush, and the Brock-Givner afterimage transfer tests are all methods of investigating for the presence of eccentric fixation. The Hirschberg test allows for the determination of the direction, magnitude, and frequency of the ocular deviation. The Bruckner test may be used to detect small angle deviations, media opacities, anisometropia, and tumors. Binocular versions allows for the determination of the comitancy of the deviation.

In addition to the meibomian glands, which other accessory glands of the eyelid secrete oil? Zeiss Krause Wolfring Moll

Zeiss The glands of Zeiss are accessory oil glands located on the eyelid margins adjacent to the base of the lash follicles. The glands of Moll are apocrine (sweat) glands. The glands of Wolfring and Krause are located deep in the fornix of the eyelids and serve to secrete a portion of the aqueous layer of the tear film.


Conjuntos de estudio relacionados

Civil Procedure Chapter 2- MBE Quiz

View Set

Beowulf Part One Quiz - First Quarter

View Set

Business Finance HW Ch 4 - Connect

View Set

Review sheet 2 The Endocrine System

View Set

Chapter 1: Leadership and Followership

View Set

6.5, 6.6, 6.7, 6.8 Vocab (Greece MiniTest#2)

View Set

Assessment of Musculoskeletal (39)

View Set

BII L.13 She Likes Red 她喜欢红色

View Set